LSAT and Law School Admissions Forum

Get expert LSAT preparation and law school admissions advice from PowerScore Test Preparation.

User avatar
 askuwheteau@protonmail.com
  • Posts: 30
  • Joined: Feb 08, 2024
|
#107058
Thank you Jeff...I appreciate your advice. I'll focus upon finishing the chapter.
User avatar
 askuwheteau@protonmail.com
  • Posts: 30
  • Joined: Feb 08, 2024
|
#107061
Jeff,

I'm currently completing the Some Train diagramming mini-drill (LRB 2023 Edition, Chapter 13, pg. 435) and ran into a challenge. Below's my reasoning below:

Legend:
  • ^s means a lowercase letter s is present over the conditional diagram which begins the some train.
    <> means double arrow
    <means single arrow
    > means single arrow
    <|> means double not arrow
Problem #4: J<K<^s>L yields the following inference according to the answer key: L<^s>J . However, my answer is J<^s>L.. Is my answer correct given that the some diagram goes in both directions?

Problem #7: S<|>T<^s>U yields the following inference according to the answer key: U<^s> S (negated...struckthrough). However, my answer is a negated S<^s>U . Is my answer correct given that the some diagram goes in both directions?

I ask because the textbook seems to give two different interpretations of how to form inference diagrams that go from right to left (pg. 433, paras. 1 & 2 juxtaposed against pg. 434, paras. 1-3)

Thanks,

Jonathan
User avatar
 Jeff Wren
PowerScore Staff
  • PowerScore Staff
  • Posts: 451
  • Joined: Oct 19, 2022
|
#107153
Hi Jonathan,

The short answer to both questions is that your inferences were correct and were identical in meaning to what was written in the answer key.

As you correctly mentioned, "some" is reversible, so the statement "Some doctors are lawyers" is identical in meaning to the statement "Some lawyers are doctors." Both of these statements mean that there is at least one (i.e. some) person who is both a doctor and a lawyer.
User avatar
 askuwheteau@protonmail.com
  • Posts: 30
  • Joined: Feb 08, 2024
|
#107177
Hi Jeff,

Thank you for explaining this. I appreciate it.

Best,

Jonathan
User avatar
 askuwheteau@protonmail.com
  • Posts: 30
  • Joined: Feb 08, 2024
|
#107178
When doing the Most train drill, a seemingly similar situation arises as with my previous inquiry re the Some train drill.

Re the Most train drill:

#4: I have as my answer J<^m L whereas the answer key has: L > ^m J. In either case, the conditional statements are saying that Most L's are J's. Is my answer incorrect?

#7: I have as my answer Negated S < ^m U whereas the answer key has: U>^m S (negated letter S). In either case, the conditional statements are saying that Most U's are not S's. Is my answer incorrect?

Get the most out of your LSAT Prep Plus subscription.

Analyze and track your performance with our Testing and Analytics Package.